Continuity of Function W.R.T. Topology Generated by a Function












0












$begingroup$


Let $(X,d)$ be some metric space, and let $x_0 in X$ be some fixed point. Denote by $mathcal{T}_0$ the topology with respect to which $x mapsto d(x_0,x)$ is continuous. My question is,




If $x_1 neq x_0$, will the map $x mapsto d(x_1,x)$ be continuous wrt $(X,mathcal{T}_0)$?




I've tried to prove that it is true but had no luck, so I suspect that it is false. However, I'm having trouble coming up with a counterexample. In term of trying to find a counterexample, I considered the function $f_0 : Bbb{R} to [0,infty)$ and the topology $mathcal{T}_0$ wrt it is continuous, and then tried showing that $f_1 : Bbb{R} to [0,infty)$ via $f(x) = |1-x|$ is not continuous wrt $(Bbb{R},mathcal{T}_0)$ by considering various pullbacks of closed sets. Sadly, I didn't have much luck. I could use some help sorting all of this out.



Going back to the setup at the beginning of this post, ultimately I am trying to prove that $mathcal{T}_0$ is finer than the metric topology. If I could show that $x mapsto d(x_1,x)$ is in fact continuous wrt $(X,mathcal{T}_0)$, then I would have an easy proof that $mathcal{T}_0$ is finer than the metric topology.










share|cite|improve this question











$endgroup$












  • $begingroup$
    I think either the old prelim has an error or you have copied it wrong. First of all, there is no such thing as "the topology with respect to which $xmapsto d(x_0,x)$". Second of all, assuming $mathcal{T}_0$ is just supposed to be an arbitrary topology with this property, you need the property to hold for every $x_0in X$, not just for some fixed $x_0$. It is not true if you just have one $x_0$.
    $endgroup$
    – Eric Wofsey
    Dec 19 '18 at 20:33












  • $begingroup$
    @EricWofsey You're right. The problem uses the indefinite article, but I am not sure I follow your second point. Here's the prelim: albany.edu/math/images/prelims/top1601.pdf
    $endgroup$
    – user193319
    Dec 19 '18 at 20:37










  • $begingroup$
    Yeah, the problem definitely intends to have a universal quantifier on $x_0$, not an existential quantifier. The phrasing with "fix a point $x_0in X$" at the start is a mistake. (It is correct for the first half of the problem but not the second half.)
    $endgroup$
    – Eric Wofsey
    Dec 19 '18 at 20:37












  • $begingroup$
    @EricWofsey Ah, so $mathcal{T}_0$ is a topology on $X$ with respect to which $y mapsto d(x,y)$ is continuous for every $x in X$?
    $endgroup$
    – user193319
    Dec 19 '18 at 20:41






  • 2




    $begingroup$
    Right, that's the intended meaning of the second part.
    $endgroup$
    – Eric Wofsey
    Dec 19 '18 at 20:42
















0












$begingroup$


Let $(X,d)$ be some metric space, and let $x_0 in X$ be some fixed point. Denote by $mathcal{T}_0$ the topology with respect to which $x mapsto d(x_0,x)$ is continuous. My question is,




If $x_1 neq x_0$, will the map $x mapsto d(x_1,x)$ be continuous wrt $(X,mathcal{T}_0)$?




I've tried to prove that it is true but had no luck, so I suspect that it is false. However, I'm having trouble coming up with a counterexample. In term of trying to find a counterexample, I considered the function $f_0 : Bbb{R} to [0,infty)$ and the topology $mathcal{T}_0$ wrt it is continuous, and then tried showing that $f_1 : Bbb{R} to [0,infty)$ via $f(x) = |1-x|$ is not continuous wrt $(Bbb{R},mathcal{T}_0)$ by considering various pullbacks of closed sets. Sadly, I didn't have much luck. I could use some help sorting all of this out.



Going back to the setup at the beginning of this post, ultimately I am trying to prove that $mathcal{T}_0$ is finer than the metric topology. If I could show that $x mapsto d(x_1,x)$ is in fact continuous wrt $(X,mathcal{T}_0)$, then I would have an easy proof that $mathcal{T}_0$ is finer than the metric topology.










share|cite|improve this question











$endgroup$












  • $begingroup$
    I think either the old prelim has an error or you have copied it wrong. First of all, there is no such thing as "the topology with respect to which $xmapsto d(x_0,x)$". Second of all, assuming $mathcal{T}_0$ is just supposed to be an arbitrary topology with this property, you need the property to hold for every $x_0in X$, not just for some fixed $x_0$. It is not true if you just have one $x_0$.
    $endgroup$
    – Eric Wofsey
    Dec 19 '18 at 20:33












  • $begingroup$
    @EricWofsey You're right. The problem uses the indefinite article, but I am not sure I follow your second point. Here's the prelim: albany.edu/math/images/prelims/top1601.pdf
    $endgroup$
    – user193319
    Dec 19 '18 at 20:37










  • $begingroup$
    Yeah, the problem definitely intends to have a universal quantifier on $x_0$, not an existential quantifier. The phrasing with "fix a point $x_0in X$" at the start is a mistake. (It is correct for the first half of the problem but not the second half.)
    $endgroup$
    – Eric Wofsey
    Dec 19 '18 at 20:37












  • $begingroup$
    @EricWofsey Ah, so $mathcal{T}_0$ is a topology on $X$ with respect to which $y mapsto d(x,y)$ is continuous for every $x in X$?
    $endgroup$
    – user193319
    Dec 19 '18 at 20:41






  • 2




    $begingroup$
    Right, that's the intended meaning of the second part.
    $endgroup$
    – Eric Wofsey
    Dec 19 '18 at 20:42














0












0








0





$begingroup$


Let $(X,d)$ be some metric space, and let $x_0 in X$ be some fixed point. Denote by $mathcal{T}_0$ the topology with respect to which $x mapsto d(x_0,x)$ is continuous. My question is,




If $x_1 neq x_0$, will the map $x mapsto d(x_1,x)$ be continuous wrt $(X,mathcal{T}_0)$?




I've tried to prove that it is true but had no luck, so I suspect that it is false. However, I'm having trouble coming up with a counterexample. In term of trying to find a counterexample, I considered the function $f_0 : Bbb{R} to [0,infty)$ and the topology $mathcal{T}_0$ wrt it is continuous, and then tried showing that $f_1 : Bbb{R} to [0,infty)$ via $f(x) = |1-x|$ is not continuous wrt $(Bbb{R},mathcal{T}_0)$ by considering various pullbacks of closed sets. Sadly, I didn't have much luck. I could use some help sorting all of this out.



Going back to the setup at the beginning of this post, ultimately I am trying to prove that $mathcal{T}_0$ is finer than the metric topology. If I could show that $x mapsto d(x_1,x)$ is in fact continuous wrt $(X,mathcal{T}_0)$, then I would have an easy proof that $mathcal{T}_0$ is finer than the metric topology.










share|cite|improve this question











$endgroup$




Let $(X,d)$ be some metric space, and let $x_0 in X$ be some fixed point. Denote by $mathcal{T}_0$ the topology with respect to which $x mapsto d(x_0,x)$ is continuous. My question is,




If $x_1 neq x_0$, will the map $x mapsto d(x_1,x)$ be continuous wrt $(X,mathcal{T}_0)$?




I've tried to prove that it is true but had no luck, so I suspect that it is false. However, I'm having trouble coming up with a counterexample. In term of trying to find a counterexample, I considered the function $f_0 : Bbb{R} to [0,infty)$ and the topology $mathcal{T}_0$ wrt it is continuous, and then tried showing that $f_1 : Bbb{R} to [0,infty)$ via $f(x) = |1-x|$ is not continuous wrt $(Bbb{R},mathcal{T}_0)$ by considering various pullbacks of closed sets. Sadly, I didn't have much luck. I could use some help sorting all of this out.



Going back to the setup at the beginning of this post, ultimately I am trying to prove that $mathcal{T}_0$ is finer than the metric topology. If I could show that $x mapsto d(x_1,x)$ is in fact continuous wrt $(X,mathcal{T}_0)$, then I would have an easy proof that $mathcal{T}_0$ is finer than the metric topology.







general-topology continuity metric-spaces






share|cite|improve this question















share|cite|improve this question













share|cite|improve this question




share|cite|improve this question








edited Dec 19 '18 at 20:19







user193319

















asked Dec 19 '18 at 19:56









user193319user193319

2,4212926




2,4212926












  • $begingroup$
    I think either the old prelim has an error or you have copied it wrong. First of all, there is no such thing as "the topology with respect to which $xmapsto d(x_0,x)$". Second of all, assuming $mathcal{T}_0$ is just supposed to be an arbitrary topology with this property, you need the property to hold for every $x_0in X$, not just for some fixed $x_0$. It is not true if you just have one $x_0$.
    $endgroup$
    – Eric Wofsey
    Dec 19 '18 at 20:33












  • $begingroup$
    @EricWofsey You're right. The problem uses the indefinite article, but I am not sure I follow your second point. Here's the prelim: albany.edu/math/images/prelims/top1601.pdf
    $endgroup$
    – user193319
    Dec 19 '18 at 20:37










  • $begingroup$
    Yeah, the problem definitely intends to have a universal quantifier on $x_0$, not an existential quantifier. The phrasing with "fix a point $x_0in X$" at the start is a mistake. (It is correct for the first half of the problem but not the second half.)
    $endgroup$
    – Eric Wofsey
    Dec 19 '18 at 20:37












  • $begingroup$
    @EricWofsey Ah, so $mathcal{T}_0$ is a topology on $X$ with respect to which $y mapsto d(x,y)$ is continuous for every $x in X$?
    $endgroup$
    – user193319
    Dec 19 '18 at 20:41






  • 2




    $begingroup$
    Right, that's the intended meaning of the second part.
    $endgroup$
    – Eric Wofsey
    Dec 19 '18 at 20:42


















  • $begingroup$
    I think either the old prelim has an error or you have copied it wrong. First of all, there is no such thing as "the topology with respect to which $xmapsto d(x_0,x)$". Second of all, assuming $mathcal{T}_0$ is just supposed to be an arbitrary topology with this property, you need the property to hold for every $x_0in X$, not just for some fixed $x_0$. It is not true if you just have one $x_0$.
    $endgroup$
    – Eric Wofsey
    Dec 19 '18 at 20:33












  • $begingroup$
    @EricWofsey You're right. The problem uses the indefinite article, but I am not sure I follow your second point. Here's the prelim: albany.edu/math/images/prelims/top1601.pdf
    $endgroup$
    – user193319
    Dec 19 '18 at 20:37










  • $begingroup$
    Yeah, the problem definitely intends to have a universal quantifier on $x_0$, not an existential quantifier. The phrasing with "fix a point $x_0in X$" at the start is a mistake. (It is correct for the first half of the problem but not the second half.)
    $endgroup$
    – Eric Wofsey
    Dec 19 '18 at 20:37












  • $begingroup$
    @EricWofsey Ah, so $mathcal{T}_0$ is a topology on $X$ with respect to which $y mapsto d(x,y)$ is continuous for every $x in X$?
    $endgroup$
    – user193319
    Dec 19 '18 at 20:41






  • 2




    $begingroup$
    Right, that's the intended meaning of the second part.
    $endgroup$
    – Eric Wofsey
    Dec 19 '18 at 20:42
















$begingroup$
I think either the old prelim has an error or you have copied it wrong. First of all, there is no such thing as "the topology with respect to which $xmapsto d(x_0,x)$". Second of all, assuming $mathcal{T}_0$ is just supposed to be an arbitrary topology with this property, you need the property to hold for every $x_0in X$, not just for some fixed $x_0$. It is not true if you just have one $x_0$.
$endgroup$
– Eric Wofsey
Dec 19 '18 at 20:33






$begingroup$
I think either the old prelim has an error or you have copied it wrong. First of all, there is no such thing as "the topology with respect to which $xmapsto d(x_0,x)$". Second of all, assuming $mathcal{T}_0$ is just supposed to be an arbitrary topology with this property, you need the property to hold for every $x_0in X$, not just for some fixed $x_0$. It is not true if you just have one $x_0$.
$endgroup$
– Eric Wofsey
Dec 19 '18 at 20:33














$begingroup$
@EricWofsey You're right. The problem uses the indefinite article, but I am not sure I follow your second point. Here's the prelim: albany.edu/math/images/prelims/top1601.pdf
$endgroup$
– user193319
Dec 19 '18 at 20:37




$begingroup$
@EricWofsey You're right. The problem uses the indefinite article, but I am not sure I follow your second point. Here's the prelim: albany.edu/math/images/prelims/top1601.pdf
$endgroup$
– user193319
Dec 19 '18 at 20:37












$begingroup$
Yeah, the problem definitely intends to have a universal quantifier on $x_0$, not an existential quantifier. The phrasing with "fix a point $x_0in X$" at the start is a mistake. (It is correct for the first half of the problem but not the second half.)
$endgroup$
– Eric Wofsey
Dec 19 '18 at 20:37






$begingroup$
Yeah, the problem definitely intends to have a universal quantifier on $x_0$, not an existential quantifier. The phrasing with "fix a point $x_0in X$" at the start is a mistake. (It is correct for the first half of the problem but not the second half.)
$endgroup$
– Eric Wofsey
Dec 19 '18 at 20:37














$begingroup$
@EricWofsey Ah, so $mathcal{T}_0$ is a topology on $X$ with respect to which $y mapsto d(x,y)$ is continuous for every $x in X$?
$endgroup$
– user193319
Dec 19 '18 at 20:41




$begingroup$
@EricWofsey Ah, so $mathcal{T}_0$ is a topology on $X$ with respect to which $y mapsto d(x,y)$ is continuous for every $x in X$?
$endgroup$
– user193319
Dec 19 '18 at 20:41




2




2




$begingroup$
Right, that's the intended meaning of the second part.
$endgroup$
– Eric Wofsey
Dec 19 '18 at 20:42




$begingroup$
Right, that's the intended meaning of the second part.
$endgroup$
– Eric Wofsey
Dec 19 '18 at 20:42










1 Answer
1






active

oldest

votes


















1












$begingroup$

Here is an answer if you read the question verbatim: consider $mathbb R$ with the usual metric. One topology which makes $x to d(x,0)$ continuous is the collection of sets of the form ${x:|x| in U}$ where $U$ is open in $mathbb R$. The function $x to d(x,1)$ is not continuous in this topology because ${x: |x-1| <1}=(0,2)$ is not of the form ${x:|x| in U}$ for any open set $U$. [ Sets of the the latter type are all symmetric about $0$].






share|cite|improve this answer









$endgroup$













    Your Answer





    StackExchange.ifUsing("editor", function () {
    return StackExchange.using("mathjaxEditing", function () {
    StackExchange.MarkdownEditor.creationCallbacks.add(function (editor, postfix) {
    StackExchange.mathjaxEditing.prepareWmdForMathJax(editor, postfix, [["$", "$"], ["\\(","\\)"]]);
    });
    });
    }, "mathjax-editing");

    StackExchange.ready(function() {
    var channelOptions = {
    tags: "".split(" "),
    id: "69"
    };
    initTagRenderer("".split(" "), "".split(" "), channelOptions);

    StackExchange.using("externalEditor", function() {
    // Have to fire editor after snippets, if snippets enabled
    if (StackExchange.settings.snippets.snippetsEnabled) {
    StackExchange.using("snippets", function() {
    createEditor();
    });
    }
    else {
    createEditor();
    }
    });

    function createEditor() {
    StackExchange.prepareEditor({
    heartbeatType: 'answer',
    autoActivateHeartbeat: false,
    convertImagesToLinks: true,
    noModals: true,
    showLowRepImageUploadWarning: true,
    reputationToPostImages: 10,
    bindNavPrevention: true,
    postfix: "",
    imageUploader: {
    brandingHtml: "Powered by u003ca class="icon-imgur-white" href="https://imgur.com/"u003eu003c/au003e",
    contentPolicyHtml: "User contributions licensed under u003ca href="https://creativecommons.org/licenses/by-sa/3.0/"u003ecc by-sa 3.0 with attribution requiredu003c/au003e u003ca href="https://stackoverflow.com/legal/content-policy"u003e(content policy)u003c/au003e",
    allowUrls: true
    },
    noCode: true, onDemand: true,
    discardSelector: ".discard-answer"
    ,immediatelyShowMarkdownHelp:true
    });


    }
    });














    draft saved

    draft discarded


















    StackExchange.ready(
    function () {
    StackExchange.openid.initPostLogin('.new-post-login', 'https%3a%2f%2fmath.stackexchange.com%2fquestions%2f3046816%2fcontinuity-of-function-w-r-t-topology-generated-by-a-function%23new-answer', 'question_page');
    }
    );

    Post as a guest















    Required, but never shown

























    1 Answer
    1






    active

    oldest

    votes








    1 Answer
    1






    active

    oldest

    votes









    active

    oldest

    votes






    active

    oldest

    votes









    1












    $begingroup$

    Here is an answer if you read the question verbatim: consider $mathbb R$ with the usual metric. One topology which makes $x to d(x,0)$ continuous is the collection of sets of the form ${x:|x| in U}$ where $U$ is open in $mathbb R$. The function $x to d(x,1)$ is not continuous in this topology because ${x: |x-1| <1}=(0,2)$ is not of the form ${x:|x| in U}$ for any open set $U$. [ Sets of the the latter type are all symmetric about $0$].






    share|cite|improve this answer









    $endgroup$


















      1












      $begingroup$

      Here is an answer if you read the question verbatim: consider $mathbb R$ with the usual metric. One topology which makes $x to d(x,0)$ continuous is the collection of sets of the form ${x:|x| in U}$ where $U$ is open in $mathbb R$. The function $x to d(x,1)$ is not continuous in this topology because ${x: |x-1| <1}=(0,2)$ is not of the form ${x:|x| in U}$ for any open set $U$. [ Sets of the the latter type are all symmetric about $0$].






      share|cite|improve this answer









      $endgroup$
















        1












        1








        1





        $begingroup$

        Here is an answer if you read the question verbatim: consider $mathbb R$ with the usual metric. One topology which makes $x to d(x,0)$ continuous is the collection of sets of the form ${x:|x| in U}$ where $U$ is open in $mathbb R$. The function $x to d(x,1)$ is not continuous in this topology because ${x: |x-1| <1}=(0,2)$ is not of the form ${x:|x| in U}$ for any open set $U$. [ Sets of the the latter type are all symmetric about $0$].






        share|cite|improve this answer









        $endgroup$



        Here is an answer if you read the question verbatim: consider $mathbb R$ with the usual metric. One topology which makes $x to d(x,0)$ continuous is the collection of sets of the form ${x:|x| in U}$ where $U$ is open in $mathbb R$. The function $x to d(x,1)$ is not continuous in this topology because ${x: |x-1| <1}=(0,2)$ is not of the form ${x:|x| in U}$ for any open set $U$. [ Sets of the the latter type are all symmetric about $0$].







        share|cite|improve this answer












        share|cite|improve this answer



        share|cite|improve this answer










        answered Dec 19 '18 at 23:44









        Kavi Rama MurthyKavi Rama Murthy

        63.9k42464




        63.9k42464






























            draft saved

            draft discarded




















































            Thanks for contributing an answer to Mathematics Stack Exchange!


            • Please be sure to answer the question. Provide details and share your research!

            But avoid



            • Asking for help, clarification, or responding to other answers.

            • Making statements based on opinion; back them up with references or personal experience.


            Use MathJax to format equations. MathJax reference.


            To learn more, see our tips on writing great answers.




            draft saved


            draft discarded














            StackExchange.ready(
            function () {
            StackExchange.openid.initPostLogin('.new-post-login', 'https%3a%2f%2fmath.stackexchange.com%2fquestions%2f3046816%2fcontinuity-of-function-w-r-t-topology-generated-by-a-function%23new-answer', 'question_page');
            }
            );

            Post as a guest















            Required, but never shown





















































            Required, but never shown














            Required, but never shown












            Required, but never shown







            Required, but never shown

































            Required, but never shown














            Required, but never shown












            Required, but never shown







            Required, but never shown







            Popular posts from this blog

            How do I know what Microsoft account the skydrive app is syncing to?

            When does type information flow backwards in C++?

            Grease: Live!